Question

Show that the language {(M1,M2): M1 and M2 are Turing machines with L(M1) is subset of...

Show that the language {(M1,M2): M1 and M2 are Turing machines with L(M1) is subset of L(M2) is undecidable

0 0
Add a comment Improve this question Transcribed image text
Answer #1

According to Rice's Theorm,  if you have a nontrivial set of languages, then the set of (codes of) TMs that accept these languages is not decidable. In this problem, however, we have pairs of languages. One trick is to specialize the problem to make it fit Rice's theorem by fixing M2. For example, fix some M2 such that L(M2) is empty. In this case, if SUBSETTM is decidable, then its subset with fixed M2 {<M1, M2> : M1 is a Turing machine and L(M1) is empty} is decidable as well. This, in turn, means that {<M> : M is a Turing machine and L(M1) is empty} is decidable.

Add a comment
Know the answer?
Add Answer to:
Show that the language {(M1,M2): M1 and M2 are Turing machines with L(M1) is subset of...
Your Answer:

Post as a guest

Your Name:

What's your source?

Earn Coins

Coins can be redeemed for fabulous gifts.

Not the answer you're looking for? Ask your own homework help question. Our experts will answer your question WITHIN MINUTES for Free.
Similar Homework Help Questions
ADVERTISEMENT
Free Homework Help App
Download From Google Play
Scan Your Homework
to Get Instant Free Answers
Need Online Homework Help?
Ask a Question
Get Answers For Free
Most questions answered within 3 hours.
ADVERTISEMENT
ADVERTISEMENT
Active Questions
ADVERTISEMENT